Đến nội dung

I Love MC

I Love MC

Đăng ký: 07-06-2014
Offline Đăng nhập: 04-03-2022 - 18:06
****-

#507505 $\sum \frac{a^3}{\left ( 1+b \right )...

Gửi bởi I Love MC trong 17-06-2014 - 20:58

Dự đoán điểm rơi xảy ra khi $a=b=c=1$ 
$\frac{a^3}{(1+b)(1+c)}+\frac{1+b}{8}+\frac{1+c}{8} \ge \frac{3a}{4}$ 
Tương tự suy ra $VT \ge \frac{2(a+b+c)-3}{4} \ge \frac{2.3.\sqrt{abc}-3}{4}=0,75$




#506837 Chứng minh rằng phân số sau đây tối giản với mọi $n \in Z$:...

Gửi bởi I Love MC trong 15-06-2014 - 12:04

Tổng của một số hữu tỉ và một số vô tỉ có thể là một số hữu tỉ không? Tổng có hai số vô tỉ có thể là một số hữu tỉ được không ?

a) Giả sử : $\frac{a}{b}+x=y$ ($y$ là số hữu tỉ,$x$ là số vô tỉ) 
$\rightarrow \frac{a}{b}-y=-x$ có $\frac{a}{b}-y$ là số hữu tỉ $-x$ vô tỉ do đó $VT # VP$ 
Kết luận ........ 
b) Tương tự.



#506141 $p(p(p(x)))$=0

Gửi bởi I Love MC trong 12-06-2014 - 21:15

Đây hình như là câu 5 IMO 2006 không biết đúng không nữa  :wacko:  :lol:




#504958 CM: $\dfrac{2n}{2n + 3}..\dfrac{2...

Gửi bởi I Love MC trong 08-06-2014 - 15:19

Câu này anh làm trên hocmai rồi nhỉ :D up lên cho mọi người xem.

$P(n)=\dfrac{2}{3}(\prod_{x=1}^{n}\dfrac{2x}{2x+3})<\dfrac{1}{\sqrt{(x+1)^3}}$
 
Với n=1:
 
$P(1)=\dfrac{2}{3}(\prod_{x=1}^{1}\dfrac{2x}{2x+3})<\dfrac{1}{\sqrt{(1+1)^3}}$ (Đúng)
 
Giả sử BĐT đúng với $n=k$ ta sẽ chứng minh nó đúng với $n=k+1$, thật vậy:
 
Trước hết ta có:
 
$9(k+1)^3(2k+5)^2-4(k+2)^3(2k+2)^2=(k+1)^2(20k^3+120k^2+213k+97) >0 ( \forall k>0)$
 
Nên: $(\dfrac{2k+2}{2k+5})^2<\dfrac{9}{4}\dfrac{(k+1)^3}{(k+2)^3} \\\Leftrightarrow  \dfrac{2k+2}{2k+5}<\dfrac{3}{2}.\sqrt{\dfrac{(k+1)^3}{(k+2)^3}} \\\Leftrightarrow \dfrac{2}{3}.\dfrac{1}{\sqrt{(1+k)^3}}.\dfrac{2k+2}{2k+5}<\dfrac{1}{\sqrt{(k+2)^3}}$
 
Nên:
 
$P(k+1)=\dfrac{2}{3}(\prod_{x=1}^{k+1}\dfrac{2x}{2x+3})< \dfrac{2}{3}.\dfrac{1}{\sqrt{(1+k)^3}}. \dfrac{2k+2}{2k+5}<\dfrac{1}{\sqrt{(k+2)^3}}$
 
Vậy BDT đúng với $k+1$ nên theo nguyên lí qui nạp ta có đpcm.



#504931 C/m rằng $MA_1^n+(-MA_3)^n+MA_5=MA_2^n+(-MA_4)^n+MA_n^6$ với...

Gửi bởi I Love MC trong 08-06-2014 - 12:22

Cho lục giác đều $A_1A_2A_3A_4A_5A_6$ nội tiếp đường tròn $(O;R)$ , M là điểm nằm trên cung nhỏ $A_1A_6$. C/m rằng 
$MA_1^n+(-MA_3)^n+MA_5=MA_2^n+(-MA_4)^n+MA_n^6$ với $n \in {1;2;4}$ 



#504914 Tìm GTNN của $P=x+y+\frac{10}{x+y}$

Gửi bởi I Love MC trong 08-06-2014 - 10:30

Tham khảo tại https://www.facebook...ntrung/?fref=ts ở  bài của Hiếu A Master.




#504910 $x^{2}+3=5y$

Gửi bởi I Love MC trong 08-06-2014 - 10:22

VT chỉ có thể là 
$VT \equiv 7 \euiv 2$ (mod 5) và $VT \equiv 4$ (mod 5) và $VP \equiv 0$ (mod 5) nên không có $x,y$ thỏa mãn.




#504907 Tìm GTNN của biểu thức: $A=\frac{2ab+a+b+c(ab-1)}{(a...

Gửi bởi I Love MC trong 08-06-2014 - 10:11

$P=\frac{1}{1+c}+\frac{ab+abc-c-1}{(a+1)(b+1)(c+1)}=\frac{1}{1+c}+\left [ \frac{ab-1}{(1+a)(1+b)}+1 \right ]-1=\frac{1}{1+c}-1+\frac{a}{1+a}+\frac{b}{1+b}=\frac{a}{1+a}+\frac{b}{1+b}-\frac{c}{1+c}$

Thật vậy, ta có $P\geq \frac{5}{12}\Leftrightarrow \left ( \frac{3}{4}-\frac{c}{1+c} \right )+\left ( \frac{b}{1+b}-\frac{2}{3} \right )+(\frac{a}{1+a}-\frac{1}{2})\geq 0\Leftrightarrow \frac{3-c}{4(1+c)}+\frac{b-2}{3(b+1)}+\frac{a-1}{2(a+1)}\geq 0$

Áp dụng phép nhóm Abel, ta có BĐT tương đương với $(3-c)(\frac{1}{4(c+1)}-\frac{1}{3(b+1)})+\left [ (3-c)+(b-2) \right ]\left [ \frac{1}{3(b+1)}-\frac{1}{2(a+1)} \right ]+\left [ (3-c)+(b-2)+(a+1) \right ]\frac{1}{2(a+1)}\geq 0\Leftrightarrow \frac{(3-c)(3b-4c+1)}{12(b+1)(c+1)}+\frac{(b+1-c)(2a-3b-1)}{6(a+1)(b+1)}+\frac{a+b-c}{2(a+1)}\geq 0$




#504833 các chuyên đề toán đại số 9

Gửi bởi I Love MC trong 07-06-2014 - 22:08

http://idoc.vn/tai-l...-toan-lop9.html trước tiên phải ôn kiến thức cho vững rồi làm nâng cao.




#504831 So sánh $2012^{2013}$ và $2013^{2012}$

Gửi bởi I Love MC trong 07-06-2014 - 22:06

Cách c/m bđt mà bạn nói trên 
Quy nạp 
Mệnh đề đúng với r=0 
Giả sử mệnh đề đúng với $r=k$ tức là $(1+x)^k \ge 1+r.k$
Cần c/m $(1+x)^{k+1} \ge 1+r(k+1)$  
Thậy vậy $VT \ge (1+x)(1+kx)=1+kx+x+x^2k=1+(x+1)k+kx^2 \ge 1+(k+1)x$




#504764 $\boxed{\text{Chuyên Đề}}$ Bất đẳng thức - Cực trị

Gửi bởi I Love MC trong 07-06-2014 - 18:57

1 số bài khó 
177) Cho $a_1,a_2,..,a_n>0$ với $n \ge 2$. C/m 
$(a_1^3+1)(a_2^2+1)...(a_n^3+1) \ge (a_1^2a_2+1)....(a_n^2a_1+1)$ ( Cuộc thi Czech-Slovanikia-Balan 2002) 
178) Cho $x,y,z>1$ sao cho $\sum \frac{1}{x}=2$. C/m 
$\sqrt{x+z+y} \ge \sum \sqrt{x-1}$ ( Iran 1998) 
179) C/m với $a,b,c>0$ thì : 
$\sum \frac{(b+c-a)^2}{a^2+(b+c)^2} \ge \frac{3}{5}$ ( Nhật Bản 1997)




#504705 $\frac{x^{3}}{\sqrt{4-x}...

Gửi bởi I Love MC trong 07-06-2014 - 13:06

ĐK: $x<4$ 
$VT=\frac{x^3+\sqrt{4-x}x^2-4.\sqrt{4-x}=0$ 
<--> $x^3+\sqrt{4-x}x^2=4.\sqrt{4-x}$ rồi đến đây giải pt :D




#504701 Cho x,y là 2 số dương thỏa mãn $x^{5}+y^{5}=x^...

Gửi bởi I Love MC trong 07-06-2014 - 12:50

Câu 2 phải là cho $a,b,c>0$ sao cho $abc=1$ 
C/m $\sum \frac{a^3}{(1+b)(c+1)} \ge \frac{3}{4}$ 
Nếu như vậy thì $\frac{a^3}{(1+b)(c+1)}+\frac{1+b}{8}+\frac{c+1}{8} \ge \frac{3a}{4}$ theo Cauchy. 
Tương tự suy ra $VT \ge $\frac{2(a+b+c)-3}{4} \ge \frac{6-3}{4}=VP$ 
Dấu bằng xảy ra khi $x=y=z=1$
 




#504697 Khoảng trời dành cho sự sáng tạo

Gửi bởi I Love MC trong 07-06-2014 - 12:32

Mình chế được cái bày này ( không biết đụng hàng ai chưa ) 
Cho $\sqrt{\frac{x}{y+z}}=a$ $\sqrt{\frac{y}{x+z}}=b$ $\sqrt{\frac{z}{x+y}}=c$ với $x,y,z>0$ 
C/m $a.b+bc+ac>2,5$




#504694 $\frac{1}{ab+a+2} + \frac{1}{bc+b+2} + \frac{1}{ca+c+2}...

Gửi bởi I Love MC trong 07-06-2014 - 12:28

$\frac{1}{a+1}+\frac{1}{ab+1}=\frac{1}{a+1}+\frac{c}{1+c} \ge \frac{4}{a+2+ab}$ tương tự suy ra điều phải c/m